site banner

The Vacuity of Climate Science

cafeamericainmag.com

There has been a lot of CW discussion on climate change. This is an article written by someone that used to strongly believe in anthropogenic global warming and then looked at all the evidence before arriving at a different conclusion. The articles goes through what they did.

I thought a top-level submission would be more interesting as climate change is such a hot button topic and it would be good to have a top-level spot to discuss it for now. I have informed the author of this submission; they said they will drop by and engage with the comments here!

-5
Jump in the discussion.

No email address required.

The deniers are just coping or contrarians.

It is very easy:

A) CO2 absorbs infrared radiation.
B) Industrialization increased its percentage in the atmosphere.

Preindustrial CO2 rate was under 300 parts per million.
2000 it was 370 ppm
2024 it reached 420 ppm

https://ourworldindata.org/grapher/annual-co-emissions-by-region

It is skewing to the last decades: half of the emmissions from 1750 to now were emitted in the last 30 years.

The amount of coal and oil we burned and will burn is stunningly large. A hundred times more every year than all vulcanism. HOW could that NOT have an effect?

Approved vaccines are effective, they stop infections at least in 95% of cases on average for 10 years. Covid vaccine was tested in clinical trials which showed strong efficiency preventing infections.

That means that covid vaccines should be mandated to everyone, young and old and they will help to almost eliminate covid via heard immunity, right?

What went wrong with this reasoning? Despite vaccination most people got infected with covid anyway. The protection was short lasting (3-4 months at most). It reduced severity and hospitalizations in elderly though. Other people especially children were not that much affected anyway.

Biology is more complicated than we could infer from a simple graph.

I have no strong opinion about global warming but I am against trivialization of this science. Too many unknowns for me. How believable are the models? How warmer temperatures will affect us? I can see both negative and positive aspects. How much would it cost to prevent that versus to adapt to the change?

HOW could that NOT have an effect?

The obvious answer is that 120ppm is really not very much -- CO2 is essentially a trace gas in the atmosphere. Would adding 120ppm of neon to the atmosphere have noticeable effects? Maybe it would, IDK -- but it's not obvious one way or the other.

How can it not have an effect, indeed?

Well, the starting point would be to demonstrate that it can have an effect, in isolated, carefully-controlled lab conditions.

That CO2 absorbs IR has been definitively shown for over a century. That this absorption and re-emission of radiation back towards a heat source causes that heat source to heat up further? Never has been, as detailed in the article.

Thus there is no experimental basis for the claim.


As to the atmosphere itself warming up due to absorbing CO2 -- firstly this is not the greenhouse effect (which is that the surface warms, not the air). Secondly, the lower atmosphere is warmed far more by conduction&convection, which 100% of the atmospheric gases participate in, than radiation, which only the IR-absorbent constituents do. Thirdly, that increased CO2 level will cause net air warming after all effects taken into account, is nowhere near certain (see: https://www.americanthinker.com/articles/2023/10/carbion_dioxide_does_not_cause_warming.html). Fourthly, the CO2 absorption is already saturated in the troposphere; any effects of additional CO2 are said to only affect higher, cooler layers of the atmosphere. Thus any effect on the surface would not be a direct effect but rather a complex, indirect effect. One might say that this means the boundary condition of the higher-up air is warmer (see earlier points if that is true), yet this comes to fifthly: the consensus is increased CO2 levels will cause the stratosphere to cool, not to warm. Thus the boundary condition at the TOA will be a cooler TOA not a warmer one.


As to differing rates of CO2 levels somehow themselves being an evidence for warming, I refer you to the article: "Looking at longer time scales calls the general theory into question: Barral 2017 shows that CO2 levels sometimes show “inverse trends” with temperature, and Easterbrook 2016 found evidence that “global warming causes increased atmospheric CO2”, and not vice-versa."


To sum: it is nowhere near as simple or certain as you claim.

Here is the absorbtion spectrum of CO2 : https://webbook.nist.gov/cgi/cbook.cgi?ID=C124389&Type=IR-SPEC&Index=1#IR-SPEC As you can see, it has low transmittance a little over 4um wavelength and between 14-16um

Compare that to the blackbody spectrum at 300K (27C, or 80F in freedom units) https://phet.colorado.edu/sims/html/blackbody-spectrum/latest/blackbody-spectrum_en.html

The emission of blackbody radiation starts at a little below 4um, peaks at 9.8um, then continues with significant emission to above 20um. so the absorbsion of the CO2 is within the ground emission spectrum.

The sun emits radiation at 5600K, much shorter wavelenght which are not absorbed by CO2, those radiation hit the ground, heat it, and the ground emits black body radiation at around 300K. that longer wavelength radiation leaves the ground. As that radiation goes up, the part of it that is within CO2 absorbtion spectrum can get absorbed. When a CO2 molecule absorbs a photon, one if it's electron gets bumped to a higher energy level, then one of two things can happen : A) the electron goes back to it's original energy level, emitting another photon in a random direction, or B) the CO2 molecule hits another molecule and the excited electron gives back it's energy to the other molecule as kinetic energy (I.E. heat). A is much more likely to happen in gases, while B is more likely in solids/liquids.

when A happens, it does not really heat the atmosphere, but it impedes the ground heat dissipation in the wavelenghts where there is high absorbtion. So if we are to keep the energy balance, what happens to the ground ? It has to increase temperature to dissipate more heat in the wavelengths that are not affected by CO2.

All of that stuff was figured out long ago in the 1800s, it's pretty baffling that people can deny it 200 years later. But I don't lose sleep over it, I took the doom pill over climate change long ago. We're fucked. even if we could magically convince everybody that we ought to do something about it, it would require a level of international cooperation that I think is impossible. How do you make all the world powers sit around a table and agree on a "let's all get poor until we figure out better energy sources" program when they can't agree on much smaller things.

and as for experimental measurement of CO2 radiative forcing, I found this paper : https://sci-hub.se/10.1038/nature14240

Preindustrial CO2 rate was under 300 parts per million.

2000 it was 370 ppm

2024 it reached 420 ppm

https://ourworldindata.org/grapher/annual-co-emissions-by-region

It is skewing to the last decades: half of the emmissions from 1750 to now were emitted in the last 30 years.

Well, that's concerning...

On the other hand, excuse me, but how am I supposed to take anyone screaming at me about global warming seriously? All the "adults in the room" are coming up with galaxy-brained plans to limit air travel and ban internal combustion engines, when the vast majority of that trend is coming from Asia? Why are we building solar farms in clouded regions of the northern hemisphere instead of chipping in for the nuclear electrification of the developing world?

"people fail at dealing with complex things, are happy to exaggerate for rhetoric and ignore claimed implications" is nothing new

galaxy-brained plans to limit air travel

only for peasants, private flights are not going to be affected

ban internal combustion engines

luxury cars are exempt


Nevertheless, you have weird people going with full denial of established physics and screaming about 2nd law of thermodynamics without understanding it, and fail to interpret simple graphs.

Because apparently failing at dealing with complex things, exaggeration for rhetoric and ignoring claimed implication are easier to do. Or more interesting?

"people fail at dealing with complex things, are happy to exaggerate for rhetoric and ignore claimed implications" is nothing new

That's the thing, I don't know if it counts as a complex problem. It's a "you're picking small high-hanging fruits, when you haven't even started picking the big low-hanging ones" situation.

only for peasants, private flights are not going to be affected

luxury cars are exempt

I think I'm radicalized enough as it is, you don't need to encourage me more.

Nevertheless, you have weird people going with full denial of established physics and screaming about 2nd law of thermodynamics without understanding it, and fail to interpret simple graphs.

Yeah, contrarianism is a hell of a drug, ask me how I know. But this is why I think the establishment was, and is, playing a dangerous game, burning trust to meet short-term goals. We're probably gonna keep getting more and more people questioning the most basic things around them.

That's the thing, I don't know if it counts as a complex problem. It's a "you're picking small high-hanging fruits, when you haven't even started picking the big low-hanging ones" situation.

"nuclear power has scary failure modes, but it is rare, does not actually kill so many people compared to failure modes of hydro and regular operation of coal burning and can be actually power our civilisation is safeteism is only of strong variety rather than extremely insane variety" apparently is a complex problem. Too many people think that ideal answer without drawbacks is achievable and anything with problems should be discarded.

Kind of "you're picking small pretty high-hanging fruits, when you haven't even started picking the big ugly low-hanging ones"

Some years ago I would have considered tackling climate change my main political priority. Since then, having watched most developed nations do what looks like everything in their power to actively avoid investing in more nuclear power, I find it hard to care much anymore.

I'm baffled by your first sentence, namely by the phrase: "tackling climate change". Yes, nuclear power is good, it gives us 24/7/365 electricity. It has a very low environmental impact. But nuclear power does not "tackle climate change". Climate change isn't caused by human energy use (or none use); it happens due to the interaction of the Sun with earth. Evidence I have for that is not annedotal, nor fossil fuel derived. It is certain. I studied this to be certain for myself; because I was watching trillions of money being thrown at renewable energy; so I thought to myself - I better be sure before I decide which side to join. Nir Shaviv gives a good climate change overview here: https://youtube.com/watch?v=SzITX46XHog

For me, the way nuclear power is handled is a dead give-away that the climate alarmists aren't actually interested in the climate. If CO2 emissions really as as catastrophically dangerous as they are made out to be, then nuclear is the obvious, guaranteed-to-work, 100% solution that would completely have already solved this problem by now.

As in the article: "This is highly relevant because it means our current climate scare is based not on irrefutable scientific evidence but rather on hysteria and alarmist fear-mongering that fifty years of “failed apocalyptic predictions” have failed to abate. This is crucial to understand as it makes it clear that rather than debating how humans should mitigate this alleged impending disaster, the proper focus should be to question why those in power are employing psychological fear tactics to promote taxation, restriction, and degrowth, and why so many intelligent people have uncritically bought into the hysteria when these proposed policies are clearly to their own detriment."

For me, the way nuclear power is handled is a dead give-away that the climate alarmists aren't actually interested in the climate.

I see it more as a dead give-away of the usual incompetence of governements. Would you argue that governements are not interested in heatlhcare and education ? Yet they manage to do stupid things in both domains. the same is true for the environnement.

They are interested in the climate, but they're interested not in terms of what concrete benefits or harms will it produce, but in whether we're doing a set of things that would be considered bad. Nuclear energy has disasters and waste, so therefore they think it's bad. They don't do a cost-benefit analysis, they just do a cost analysis (for things they don't like) and a benefit analysis (for things they do).

They do genuinely care about the climate, but usually in a scattered rather than coherent way, I suppose.

(Of course, not all climate activists, etc.)

Many climate activists are very much in favor of nuclear power, myself included. Sometimes it's a matter of going with the flow and allocating your advocacy resources where you think the'll make the most difference. Nuclear is a hard sell for a lot of people, and getting new plants open will take a long time mainly due to regulatory barriers -- which have to change before you can even start constructing. Solar and wind are seeing a lot of growth right now, and while they don't solve the baseline load problem, will make a dent in the issue in the meantime.

I don't disagree that many people who label themselves "climate activists" are irrationally fearful of nuclear, but they are not as representative of the entire group as the article.

If CO2 emissions really as as catastrophically dangerous as they are made out to be, then nuclear is the obvious, guaranteed-to-work, 100% solution that would completely have already solved this problem by now

Unless the same people also fear nuclear power to roughly the same extent. And unfortunately many people who drive environmental concerns grew up in an era where fear of nuclear power was rampant. The Cuban Missile crisis, Chernobyl, Three Mile Island. In other words if you think A will be a catastrophe and can be solved by B, which will also be a catastrophe then it becomes easy to see why people look for options C through F.

The real test is once those people die/retire/age out of leadership roles will the movement reorient itself.

Same as generals still fighting the last type of war instead of the next one.

Notably, they can of course be wrong about how catastrophic A or B might be, but from direct exposure to very many high level "climate alarmists" it is my opinion they are absolutely sincere in being worried about the climate. They are just also worried about nuclear catastrophes. And a whole bunch of other things. In fact I would say the thing that connects them (or most of them), is they worry way too much about a lot of things.

After all if this fear of climate change is driven by hysteria, what makes you think their other fears are going to be rationally evaluated against climate change in order to solve climate change?

what makes you think their other fears are going to be rationally evaluated against climate change in order to solve climate change?

Or perhaps more generally, what makes you think they're even capable of rationally evaluating fears in the first place?

it is my opinion they are absolutely sincere in being worried about the climate

Well, that's the good-faith answer. Yet, it concerns me that the things they appear to be genuinely afraid of also happen to be things that it is in their personal or class interest to be genuinely afraid of, and afraid in such a way that their opponents' good-faith efforts are never good enough for them.

If climate change sheds its master morality baggage and actually threatens to improve life for a change, maybe we'd start accomplishing those goals. Tesla did it, look how successful they are. (Of course, the most statistically worried about climate change also excuse themselves from buying a Tesla because they don't like what Elon says on the Internet- a good faith view of that is hyperconservative fear paralysis... which is why it's odd we consider progressives to be on the left when they're fundamentally an ultraconservative rightist movement specifically because fear dominates their reasoning.)

Some of the alarmists are explicit about their ideological tendencies, see: https://twitter.com/ClimateBen/status/1766859556313841773

"Does capitalism mean fully entering rapid mass extinction in either the 21st or 22nd century?

Yes.

Will humans and the majority of species face mass death or annihilation by 2050- 2150?

Yes.

Are mass media journalists telling us scientists urge economic system change?

No."

And later in the thread: "Abrupt climate change is just one compounding factor in extinction catastrophe. Change this Extinction Economy now while it's still too late to protect species and everyone."

I suspect there's a large overlap between climate alarmism and anti-capitalism.

This is highly dangerous. As the image shows (source: https://ourworldindata.org/extreme-history-methods), the global poverty rate was over 80% in 1800. Today it's less than 20%. Capitalism and the industrial revolution it helped to fuel is the obvious cause of that. Note that modern-day class warriors rail against the inequality of capitalism -- but they leave out the fact that the vast majority of people on the planet are far better off today than 250 years ago.

The dangerous aspect is if they have their way, the systems that enabled this to happen will be reversed and thus we'll head back in the direction of extreme poverty for all.

/images/17133444819677088.webp

but they leave out the fact that the vast majority of people on the planet are far better off today than 250 years ago.

vast majority is underestimate, people worse off are tiny % of all population. And they are mostly in places like North Korea where it is caused by not enough capitalism. Or Ukrainian frontline where it can be blamed for many things but surely not capitalism.

Or perhaps more generally, what makes you think they're even capable of rationally evaluating fears in the first place

Exactly. People aren't generally rational, especially about their fears. And especially not people who have been shown not to be rational about their fears. So condeming them for not rationally comparing fears and thus saying they are operating in bad faith is just double dipping.

Fighting climate change and nuclear catastrophe are not about improving lives. They are about stopping them getting worse (or ending entirely!). Teslas are nice..but they are still (in my view) inferior to a similary luxurious petrol car. If climate change is a real problem, then the deal may be tackling it at a cost. Lives may not get better. It entirely depends on how bad it will be and what the cost would be. Maybe we would have to spend 10% of world GDP on some huge geo-engineering project INSTEAD of making peoples lives better in the short term.

Now, are people hypocrites? Do they struggle to make the sacrifices their principles tell them they should? Do they fail to rationally compare their dislike of Musk to whether his cars help their ideals? Yes to all of those. So it goes.

If climate change is a real problem, then the deal may be tackling it at a cost.

Hence "just build nuclear plants; if you thought it was such a problem you would already have accepted the added risk".

So either climate change isn't actually the existential risk they claim because they're willing to let other hysteria take precedence, or they are correct about it being the most important existential risk... which means the environment is precious enough that we're willing to let a reactor mess up a city or two. Drop in the bucket compared to "the world will be destroyed".

Teslas are nice..but they are still (in my view) inferior to a similary luxurious petrol car.

I wholeheartedly agree, but the reason given for not buying one is not "Electric car inherently inferior", it's "Elon man bad". People who take steps in solving the problem should be honored among those with the grievance; that is historically how the people with the grievance pay for the solutions. That they refuse to pay now, and will do whatever they can not to pay (the person who has done more for Blue environmental goals in decades than anyone else... is also their biggest political target), is notable.

Hence "just build nuclear plants; if you thought it was such a problem you would already have accepted the added risk".

Unless you also feel nuclear catastophe has a similar cost. Remember we've already established the risks are not being evaluated rationally. So you can't then use the fact they are not evaluating the comparative nuclear risk rationally as evidence of anything other than irrationality, thats what I mean by double dipping.

Remember these are not utilitarians. Just like the answer as to why groups who feel like abortion is a Holocaust happening every year aren't concentrating solely on that. That is just how people are. Virtually nobody who claims to believe that X is the worst thing in the world are willing to trade off other bad things against it in a rational manner. It is just not how we operate by and large. We are not rational. They are not rational. Rationalists are not actually rational (although they try).

So people who try to use that against a group (whether that is claiming Christians don't really believe abortion is murder or that climate worriers don't really believe in climate change), are just missing the point. They do believe it. And yet they will not act as if they do. Because they also believe in many other things and are not evaluating the trade offs in a utilitarian way. So that isn't evidence of anything except that they are humanly flawed (or gifted, if you think utilitarianism is evil).

Maybe it is true that every anti-abortion advocate should quit their job and advocate full time for a federal ban given as they believe that hundreds of thousands of innocent children are being murdered annually. Maybe everyone who believes climate change is an existential threat should be crowd funding atomic reactors. But that just is not how most people work.

It is quite ironic that one of the biggest contributes to CO2 emissions reductions is fossil fuel companies fracking so aggressively they drove the price of natural gas negative at some points. Ultimately substituting NG for coal is probably a net benefit, but I'm pretty sure environmental activists are not happy with the growth of NG as a energy source.

It's been more than 15 years since An Inconvenient Truth came out and the IPPC won their Nobel Peace Prize. In that time it would have been totally possible to replace essentially all electricity production in developed countries with GEN III+ nuclear plants and make substantial progress on Gen IV plants. Instead, without utility scale storage, the focus on growing interment renewables has only entrench NG peaking plants as the dominant on-demand electricity generation source.

I am curious, have they bothered to explain why temperature is growing? Or at least leave it as an open question?

Or are they denying that Earth gets warmer?

In normal sensate reality, heat only flows from hot to cold, but the greenhouse effect appears to involve an inverted heat flow within this system.

Nope. It is about how efficient energy transfer from Sun (hot) to Earth (cold) is and how it happens. Second law of thermodynamics is not violated.

This is an article written by someone that used to strongly believe in anthropogenic global warming

This is not a strong credentials at all.

Yes. EVERYONE agrees that earth's climate gets warmer, and cooler over time. Since the late 1970s, the trend has been warming. The disagreement is about : a. causes of that warming b. amount of the warming

a. A few years ago I think most skeptics accepted some role for greenhouse gases in earth's climate change. Today, I think most skeptics accept hardly any role for greenhouse gases. They skeptic case changed for at least 3, or 4 reasons. These reasons are:

  • skeptics listened to Tony Heller who was essentially auditing what authorities said about temperature changes. Official bodies have been replacing metered readings with model readings by applying a mathematical technique called homogenization. Recently mainstream climate scientists, increasing accepted Heller's claims regarding data corruption by the authorities. By deliberately corrupting data to scare-monger, the self-styled "climate consensus" burnt their credibility.
  • Other skeptics wrote better basic models of atmospheric behaviour to explain the so-called greenhouse effect without relying on the radiative action of CO2, H2O(g) or other so-called greenhouse gases. For example I'm thinking of the Zeller/Nikolov model using adiabatic warming to explain the Lapse Rate. Also - the work of the two Connelly's - who looked at real atmospheric behaviour (results from 20 million weather balloons) but could not find a greenhouse effect in the data they looked at.
  • the data improved. All of that improvement is associated with work done by skeptics. Climate alarmists gave us no better data - they only provide us with worse models. We know far more about other planets and moons in the solar system, and our understanding of the sun's affects on earth have vastly improved.
  • the old excuses given to us by climate alarmists for never validating their models no longer wash. Climate alarmist "scientists" could once get away with excuses such as "we cannot validate the greenhouse gas model because that can only be done on a planetary scale, and we only have one planet". That does not wash because their basic model which they swore was "settled science" is now blown to pieces. None of them will defend their settled science model in public because it's a garbage model, and they don't want to be laughted at.

So the increasing success of climate skeptics in not ONLY due to the failure of official science over COVID lockdowns, vaxxes and the cover-up of the lab leak. Climate skeptics today are more unified on what we agree on, and more certain that EVERYTHING the so-called consensus say about the climate is wrong.

Urban albedo changes are small but skew temp measurements. I am more inclinde to consider agricultural albedo changes as a darkening of 11% of all land for around 4 months of the year is in line with observed warming and in a similar trend. This also explains the warming affecting the Northern Hemisphere more than the southern as is claimed in the observations. This is also in line with winters becoming milder, as dark tilled and usually wet land has a high absorbtivity compared to grassland or other forms of pasture.

I seem to recall that some scientist recently showed that heating was largely an artifact of urbanization and location of temperatures. When corrected, there is heating but much smaller amount.

When corrected, there is heating but much smaller amount.

So, assuming that all claims made in this comment are true: there is still warming anyway.

Climate warming and cooling is normal because earth NEVER had a "stable climate". Stable climate is a gaslight and lie sold to us to get us onboard with the elite's renewable energy and decarbonziation agenda.

I mentioned D-O events. See diagram. This data shows earth's climate is pretty unstable - not stable. We cannot explain these because the D-O data shows sudden climate warmings. Greenhouse gases cannot explain such sudden warmings. I could attempt to explain sudden coolings (and subsequent rewarming). No one can explain sudden warming (with subsequent cooling).

/images/17132661705570595.webp

"there is still warming anyway" <- and cooling. The climate is always changing. Climate alarmists befuddle us by telling us we should have a "stable climate" when we reach net zero. They're lying. We will never have a stable climate. Over the past 2.5 million years there's been both long term and short term instabilities. For most of the past 2.5 million years earth has been in an Ice Age - with both poles heavily glaciated for most of the time. At least 20, dramatic short-term changes known as : D-O oscillations can also be seen in the climate during the last deep glaciation. In some of these oscillations parts of the Northern hemisphere warmed and cooled by up to 3C over a few decades. We can't explain such D-O events in the climate past; but NO ONE claims they are due to greenhouse gas changes.

PS: D-O means "Dansgaard-Oeschger"; named after the two scientists who discovered them.

On a significantly smaller scale suggesting that the models relied upon to suggest a problem are just fundamentally broken.

Your curiosity can easily be satiated by... reading the article :)

I tried, but from part when author apparently fails at understanding second law of thermodynamics (or thinks that Earth during summer is hotter than Sun) I started skimming.

Can you point me to relevant part if I asked about part I missed?

It appears you started skimming earlier than that, as this is in the 3rd paragraph: "Worst of all, the alleged recent warming trend that is said to confirm earlier model predictions is based on data that appears to be adjusted to match the very models it is meant to be independently corroborating." It is elaborated in the article on the "Correlation is not causation" section.

As to the 2nd Law: "As to the thermodynamics, the arguments are plentiful. I'll just point out two physicists believed that it does violate the 2nd Law and published a peer-reviewed paper to that effect (Gerlich & Tscheuschner). Most others, of course, disagree. The point in the article is that rather than debate it, let's demonstrate it experimentally, in the real world - and this has not been done for the GHE."

As to thinking the Earth during summer is hotter than the Sun... genuinely not sure what you're addressing here.

As to the thermodynamics, the arguments are plentiful. I'll just point out two physicists believed that it does violate the 2nd Law and published a peer-reviewed paper to that effect (Gerlich & Tscheuschner)

That is not giving any info whatsoever what the claim is - and if we are going via ad authority arguments then surely you are aware that you are going to lose here? And appeal to peer-reviewed papers is not going to help you much?

Why supposedly GHE would violate second law of thermodynamics?

@dale_cloudman thinks that 2LoT means "heat flow from cold to hot is zero" rather than the correct "heat flow from cold to hot is less than heat flow from hot to cold such that net local flow is from hot to cold". It's a reasonably-easy misunderstanding to make (at least, for someone trying to make sense of a topic without the proper grounding), since when you're dealing with conduction or convection there's no separation between forward flow and back flow, and non-scientists don't deal with radiative heat transfer often.

The reason to point out that two physicists published a peer-reviewed paper where they argue the GHE violates the 2nd Law of Thermodynamics is to show that it's not a layperson's mis-understanding that underlies this claim. Trained physicists with extensive experience precisely with thermodynamics, made the argument. So it's just not effective to say it's because of lack of "proper grounding".

The terminology can be tricky. For a detailed explanation I refer readers to the paper in question, https://arxiv.org/pdf/0707.1161.pdf, p75-79 in particular.

Trained physicists with extensive experience precisely with thermodynamics, made the argument.

Not sure why you assume that scientists cannot ever be hilariously wrong, make stupid mistakes, engaging in harmful conspiracies, troll or be insane cranks. Also in their own field.

It is especially confusing given that you are claiming that massive number of scientists do at least one of this things or similar.

Mix of conspiracy theory about scientists engaging in conspiracy AND appealing to authority of scientists is weird. Can you decide on doing one of these?

"two scientists published paper about X" is not as strong argument as you think it is. Have you heard about replication crisis?

Well it appears my strategy worked. The critique went from "reasonably-easy misunderstanding to make" for someone "without the proper grounding" to: scientists can still be wrong. Mission accomplished.

More comments

The paper's authors seem to be making the same mistake and/or a slightly-different one. The slightly-different mistake is to prove that the atmosphere can't warm the ground on net (true enough on a global scale, though there are local exceptions) and then assume that this means the ground can't warm because of atmospheric effects (AGW is strictly-speaking a case of the atmosphere and radiation-to-space cooling the ground less, and while 2LoT does indeed forbid the atmosphere or space net warming the ground it says nothing about the rate at which the cooling occurs; the ultimate source of the energy that warms the planet is of course the Sun).

The obvious analogy here is that a blanket can't warm you up - it does not generate or actively transfer heat - but you get warmer when you are covered in one because it reduces the rate at which the environment cools you and thus you get net-warmed by your metabolic activity.

Yes, I know the authors go around claiming that lots of people are wrong, some of whom say things similar to this. These claims are a mixture of straight nonsense and cherry-picking people who did indeed fuck up in either their understanding or their exact wording. It's not like thinking AGW is real is an infallible defence against being a moron or messing up a description, after all.

As for their supposed training: well, they're making script-kiddie mistakes. I legitimately don't know whether the reason they're making script-kiddie mistakes is that they're script-kiddies themselves or that they're deliberately lying to fool script-kiddies like you (to be clear, I'm a script-kiddie about a lot of things; no offence intended). I'm not sure it especially matters.

I came here to post something similar. The short version is, while the article author says:

In normal sensate reality, heat only flows from hot to cold, but the greenhouse effect appears to involve an inverted heat flow within this system.

Heat DOES flow from cold to hot, it just must be less than the heat flowing from hot to cold, and that is what the referenced diagram shows.

More comments

A better article is here. It's the only one you'll really ever need. Attacking the impacts (i.e. the people hyperbolically claiming we'll all boil alive or drown or die in some other way is a lot easier (and probably truer) than saying the greenhouse effect doesn't exist.

"the greenhouse effect doesn't exist" Was easy for me to say because I spent years looking for evidence for it but finding nowt. Yes there are models; but today, I'm confident no greenhouse gas model has any credibility. The fact not one climate alarmist "scientist" will debate non-validated models they say are settled science is all the evidence I need to know their claims are fraud. Here's some evidence I collected against the greenhouse effect: https://greenfallacies.blogspot.com/2024/03/theres-no-greenhouse-effect.html | https://greenfallacies.blogspot.com/2021/10/greenhouse-gas-effect-is-junk.html | https://greenfallacies.blogspot.com/2020/10/destroying-greenhouse-gas-conjecture.html

But the greatest point I can make is: none of its supporters will defend it in open debate. That's how I know it's not mere groupthink, fallacy, nor a mere mistake. It's fraud.

Sorry, this one's more complicated than it looks.

Basically, one of the more core conceits of hardcore SJ is that debate is useless because people are too stupid to tell truth from lies, and so the correct policy (as they see it) is not to debate their opponents but to shut said opponents out of the debate hall.

Due to this conceit, SJers refusing to debate is not actually much evidence of fraud, because they do this even when they believe what they're saying.

(I'm not defending this conceit; you need debate in order to orient your understanding to the truth, and without it society falls into ideological rabbit-holes. I think this conceit of SJ is highly destructive. I'm merely explaining it.)

The world is warming and the climate is shifting. Not catastrophically, so far. They just bumped up all the growing season maps for the USA, farmers don't make shit like this up with money on the line. Here in Maine winters are becoming warm and wet, the bays don't freeze up in the winter anymore. Lobsters have all but disappeared from the NE States south of us and NY state and are slowly moving into slightly cooler Canadian waters. We've had more 100 year floods in the last few years than in the last 100. It IS changing, very rapidly on a geological time scale, it doesn't really matter if it is human caused or not; we should stop it.

I'm a big proponent of climate engineering or "geoengineering" . Our whole world is already shaped by humanity and our impact on it, I see no reason why altering the climate on purpose instead of on accident is so much worse. We should start with sulfur now, because it is cheap, we know it works, and how it works and that is is safe. Move on to space based shields later if it is still required. As many of you may have noted if you were in the path of the eclipse, no one would ever notice a 1% drop in sunlight.

Basically, climate change is a solved problem. If it ever gets bad enough we'll do something about it, I hope we do it sooner, I want my winters back.

Add ocean fertilization and enhanced weathering to your list, assuming future research doesn't have any surprises and they really are reasonable ways to sequester carbon in the medium-term and very-long-term respectively. Even if plants don't notice a drop in sunlight (or they notice but are happy enough about the extra CO2 to be fine anyway), we're already about 20% of the way from pre-industrial CO2 levels to "people complain about stale air and drowsiness" CO2 levels outdoors, and indoor air relies on CO2 diffusion to outdoor...

But frankly I'd wait before starting anything. In the US the biggest obstacles to doing anything are: (1) about a third of voters don't think climate change isn't yet causing any harm and (2) phasing out fossil fuels is going to be a massive challenge, both economically (it will be a third phase of history, where the first two were "underpopulated and dirt poor" and "burning fossil fuels") and technologically (we need a massive expansion of fission and/or massive improvements in grid battery costs), so we're probably going to need a stronger political consensus. If we go for the geoengineering too early then that consensus is always going to be split between "we fixed it" and "no, the priests just sacrificed a virgin and pretended that that's what made the drought end".

Add ocean fertilization

Ocean fertilization relies on capture in the shells of diatoms as carbonate, which is much more of a stretch than presented.

Yeah, "assuming future research doesn't have any surprises" was a predicate here, not an actually-safe assumption. Sure would have been nice if we hadn't stopped the research a decade ago.

I always thought one of the paradoxes of climate science was that (1) climate modeling is sound enough to project far into the future and determine magnitude, causality, and predict ecological, social, and economic impacts. And (2) geoengineering would be too dangerous because we don't know what the long term effects will be. That's probably not the exact phrasing of the IPCC or other consensus positions, but I don't think it's unreasonably far off either. Very speculative, but I suspect some of the skepticism of climate activism is that the solution always seems to be more socialism, rather than we would like to spend 0.005% of GDP to spray some calcite into the stratosphere.

David Friedman had a post on it where a simple regression modeled the observed phenomenon much better than the complex IPCC models.

It's more than a little suspicious to me that every solution to climate change is something the left already wanted to do for other, unrelated reasons. What a coincidence that to fight climate change we have to become vegetarians, have fewer kids, demolish the suburbs, redistribute wealth to minorities and abolish capitalism.

It's more than a little suspicious to me that every solution to climate change is something the left already wanted to do for other, unrelated reasons.

Is this true? Many of the proposals you identify with left wing now-more-than-everism emerged as reactions to climate change and other environmental problems. Others are of questionable popularity even on the far left (anti-natalism and mandatory vegetarianism are distinctly marginal position). Beyond that, you have boring centrist proposals like carbon taxes and emissions regulations (in fact, these are prone to being vilified by far left activists for being insufficiently radical, probably because they're more likely to be implemented).

In any event, it seems of little surprise that most proposed solutions for climate change are left-coded when the standard right-wing position is that climate change either isn't real or isn't a problem. The argument takes place almost entirely between the center and the left because the right refuses to participate.

Vegetarianism/Veganism has already been extremely popular on the left due to animal sympathy, and they can be quite pushy about proselytising. Mandatory Veganism is imo a weakman. Anti-natalism is the same; Having less kids has been quite popular on the left (arguably in general) because it means less obligations, more money you can spend on hedonistic pleasures, more time to do whatever you want. In both cases, climate justifications have come long, long after people argued for & adopted the change in the first place.

Also disagree on the second point. If you're actually seriously trying to tackle a problem, you'll usually end up with some technical, politically agnostic solution. If I notice that a certain widely used statistical measure is biased by, say, base rates, then I'll just recalculate it with a correction term, write a proof that the correction term indeed does what it should and maybe write a paper about it. I don't advocate that more BIPOC representation will somehow solve it (well, maybe I'll advocate for more statisticians, but that's not considered political yet). If engineers notice a turbine having a rare but potentially dangerous unexpected failure mode, they'll add a component to compensate or re-design it.

Vegetarianism/Veganism has already been extremely popular on the left due to animal sympathy

The bare minimum requirement to even begin considering the statement "vegetarianism/veganism has already been extremely popular on the left" truthful would be a majority of leftists being vegetarian or vegan, which isn't even true here, where these things probably have a stronger hold on left consciousness than most other countries, and hasn't been remotely true in any of the other countries I've visited and where I've encountered leftists.

FWIW, a large part, possibly majority but at least close to 50%, of our college-educated left-leaning friends (and it's not even unpopular among our non-college-educated friends) is some kind of vegetarian. Among those who aren't, the majority is constantly stressing how little meat they're eating. The line between them is pretty fuzzy, since there's a decent number of people who claim to not eat meat at home, but sometimes outside when there's no other option, and these people will sometimes consider themselves vegetarian anyway, sometimes not. Almost nobody is an unabashed meat eater. As justifications go, animal sympathy is at the top for the stricter vegetarians, health benefits for the less strict (this actually includes myself), climate considerations are generally second line ("and btw it's also good for the climate I've heard").

Surprisingly, this did not greatly change when we became parents; Yes there's very few super-militant vegetarian parents, but we know multiple families where only the children eat meat, not the parents, and eating relatively little meat is actually the norm.

All of those positions predate global warming as a salient issue. The Population Bomb was written in 1968. Orwell complained about vegetarians taking over socialism in the 1940s. Sociologists have been slagging car centered suburbs since they were built after WW2.

in fact, these are prone to being vilified by far left activists for being insufficiently radical

Right, because the activists (the people who matter) have more important goals than cutting emissions.

Right, because the activists (the people who matter) have more important goals than cutting emissions.

As a counterpoint, one of the biggest, most effective climate change activism groups (Citizens Climate Lobby) focuses almost exclusively on practical policy to cut emissions, mainly a revenue-neutral carbon tax.

Going back to the root though, with things like geoengineering -- I'm not 100% against it, but I'm much more in favor of addressing the root cause, rather than trying to put a band-aid on it. First issue, and maybe the biggest, is the moral hazard -- if you start geoengineering, that means countries won't try as hard to reduce emissions, immediately negating some of the benefit of geoengineering. Second, specifically for putting sulfur into the atmosphere to reduce solar irradiance, you don't get to control where that goes. It wanders all over the place, changing weather patterns, possibly causing storms or droughts. Third, cooling the earth but leaving CO2 levels higher doesn't solve ocean acidification.

I suspect that we'll need to do it at some point, but I think it is best to push hard on reducing emissions first and foremost. And maybe I partly believe this because I think that practically speaking, we can do it without a substantial reduction in living standards, if we start using carbon taxes effectively, and streamline nuclear regulations to the point where it's actually viable again.

"Never waste a crisis."

The solution to a crisis is always that thing you wanted to do anyway for unrelated reasons. That much just seems like human nature.

More effort, please. We do not allow "I agree!" or "This" or "Nuh-uh" posts.

I think one of the better arguments against geoengineering is that I don't trust the geoengineers to remain aligned with what works out best, but will likely end up with internal incentives which could possibly lead to a dramatically messed up climate. You could easily imagine people spending too much to cool the earth, if the incentives were such that that were high-status or otherwise rewarded behavior.

That said, it's probably worth attempting anyway, if we're going to be trying to mitigate anthropogenic climate change (assuming the article here is wrong and that's a thing), as it's so much cheaper. Just, it'll require care in how it's set up.

We are already geoengineering the Earth via agriculture. Apparently it occupies around 11% of the land surface and lets see what we did to that land. In general before agriculture that land was pasture or forests, which have their complex ways to mitigate heat absorption and emission throughout the year. We replaced those with arable land, which for at least 4 months of the year sits barren tilled and mostly wet. This resulted in around 3% of the Earths surface absorbing around twice the usual amount. This is significant and is in line with observed warming trends of the winters in the Northern Hemisphere. If this is the case then the current warming is anthropogenic, not really problematic and will plateau at some point. This can also be mitigated by adjusting tilling practices, but the question arises, do we want to?

By that logic, how do you trust anyone to do anything? Stuff still needs to get done, even if the incentives aren't perfect. The world turns.

By that logic, how do you trust anyone to do anything? Stuff still needs to get done, even if the incentives aren't perfect. The world turns.

You might have noticed these days we (I) don't trust the media and authorities to tell us the sky is blue today. I seriously don't trust these people not to fuck it up and engineer an even greater climate disaster.

That is your prerogative, things still happen.

Most actions don't have externalities at that scale.

If you over correct you just stop, it requires constant pumping into the upper atmosphere. A snowpiercer apocalypse isn't even possible.

It takes years for it to come down, and some of the possible failure modes can't wait years (e.g. crop failure).

Well you don't go crazy with it all at once, you add a little more till you get the climate you want. If we were on on the edge of crop failure we would pull back.

We're doing the same experiment now, only with zero controls or plans, we may as well take charge.

GHG has nothing to do with incoming shortwave; the only crop-relevant effect is temperature. I don't object to longwave geoengineering such as, y'know, air capture or olivine beaches; that's bounded to stuff we're fucking with anyway, as you say.

I object to shortwave geoengineering via aerosols and such, because there are other effects than temperature and some of those could have dire consequences. Almost everything in the Earth system comes back to sunlight in one way or another; you fuck with it at your peril.

More comments

Ah, good point that it'd go away before long.

My concern, I guess, would be that people wouldn't stop.

It's not exactly like the goals of climate activists is to achieve some socially and environmentally optimal level of fossil fuel usage, so I see no reason why we'd expect people to self-regulate here.

That said, you're right that it going away makes this unlikely to be too much of a problem.

I think we've seen after 80 years of atomic brinkmanship that most countries aren't actively suicidal, if you started getting below average temps and massive problems from cold weather the relevant authorities would stop the pumping, regardless of what the "coldies" are screaming about; maxing the temp drop to protect against future bad actors etc...

And if the relevant authorities are all "coldies"?

More comments

If “more socialism” is a good descriptor for most climate interventions, it’d apply just as well to spraying calcite. Or to building highways, or to national defense, or any number of other collective-action problems. Clearly some of these are judged to be legitimate.

One could flip the argument by observing that communists just love their mass ecological interventions.

I was more referring to statements like (from the IPCC Climate Change 2023 Synthesis Report pp. 101)

Redistributive policies across sectors and regions that shield the poor and vulnerable, social safety nets, equity, inclusion and just transitions, at all scales can enable deeper societal ambitions and resolve trade-offs with sustainable development goals.

I'm not even saying it's wrong, but putting redistributive policies ahead of mechanical interventions is what I would argue could be perceived to be driven by a political agenda. That perception can erode trust in the institutions advocating for those interventions, even if the "Equity and Inclusion" and "Scientific Basis" sections are not logically dependent on each other.

I certainly was not making the argument that general public sector works are always bad. Even for things like highways though, there are clearly different strategies that spread the costs and responsibilities differently. For example, the primarily toll-based privately maintained and operated Autoroutes in France vs the free at the point of use Interstate Highway System in the US.

You can do it ancap style with https://makesunsets.com/

That's more or less what I was gesturing at with 0.005% of GDP.

Though I would prefer they not exhaust the worlds supply of helium to do it. It's a very usefully industrial gas, and basically not renewable. Using hydrogen balloons would be much more "sustainable," though not supper feasible to do at small scale. Using their cost projections you actually get ~0.05% of gross world product per year. I assumed you could get the cost down with economies of scale by using some mix calcite substituting for the sulfur and hydrogen substituting for the helium.

I suspect some of the skepticism of climate activism is that the solution always seems to be more socialism

Undoubtedly some of it is that, but it seems likely that the primary driver is the aggregate weight is economic interests and cultural/lifestyle preferences.

A technocratic-capitalistic response that involves stopgap geoengineering + carbon taxes + a transition to net zero emissions is still going to mean a shift from policies that privileges fossil fuel usage, car-centric lifestyles, and low-density development to policies that penalize them. This is bad news if you work in the fossil fuel industry (or just live in a state where the fossil fuel industry is a major economic engine) or like car-centric lifestyle and low-density development. For that matter, it's bad news if you like beef.

Reading through the article I quickly found a couple errors in reasoning. Also overall the writing style doesn't give the impression that the author is deeply knowledgeable in the subject (or in heat transfer generally). Mostly this is unconvincing to me.

Figure 7.2 provides a helpful visual representation of it, and here we see something a bit odd. The total energy flowing out of the surface is more than three times larger than the amount flowing into it from the sun.

The author gives insinuates that this somehow violates the laws of thermodynamics, but it doesn't. I can't tell if he doesn't understand this or is being intentional misleading.

However, the described mechanism is rather puzzling. In normal sensate reality, heat only flows from hot to cold

Take two black body radiatiors at different temperatures and places them near each other. Would you say that the lower temperature body radiates in every direction /except/ the side facing the hotter? Of course not. While the net heat flow will be from hotter to colder, it is totally reasonable to talk about energy transfer from colder to hotter as a matter of accounting.

we can conjecture what one such proof [of the greenhouse effect] might look like: it would have to consist of an external energy source – such as the sun or a heat lamp – that is set to warm a surface. The energy input should be measured and the surface, in the presence of greenhouse gases, should get much hotter than that input alone can provide, emitting much more energy in response.

For 2263 w/m2 solar irradiance, 0.7 albedo, 0.85 emissivity, Venus "Should" only have a surface temperature of less than 200c, instead of the 400+c we observe. Internal heating of Venus appears to be negligible (10s of mW/m2 at the surface).

Venus can be explained by a thicker atmosphere and thus a larger adiabatic lapse rate effect. Also see: https://www.themotte.org/post/960/the-vacuity-of-climate-science/203479?context=8#context . It's just not a good demonstration of GHE.

As to the thermodynamics, the arguments are plentiful. I'll just point out two physicists believed that it does violate the 2nd Law and published a peer-reviewed paper to that effect (Gerlich & Tscheuschner). Most others, of course, disagree. The point in the article is that rather than debate it, let's demonstrate it experimentally, in the real world - and this has not been done for the GHE.

Also see: https://www.themotte.org/post/960/the-vacuity-of-climate-science/203479?context=8#context . It's just not a good demonstration of GHE.

text at this link fails to explain why supposedly Venus is not

Venus can be explained by a thicker atmosphere and thus a larger adiabatic lapse rate effect

How it would increase temperature without GHE?

Your linked post mentions nothing about adiabatic lapse rate nor how it can explain Venus' temperature being much higher than would be predicted from blackbody equilibrium. Care to explain in detail what you mean?

The linked post is meant to show that using Venus to corroborate a model is foolish, as that same reasoning was used to predict it had the same temperature as Earth. It provides just as much support to any more modern theory, namely, none at all.

For an adiabatic lapse rate on Venus description see: https://youtube.com/watch?v=_4KG0-2ckac ,

as that same reasoning was used to predict it had the same temperature as Earth.

Yeah, and Lord Kelvin estimated age of Sun to be about 32 million years (IIRC). Noone claims that scientists are always right.

For an adiabatic lapse rate on Venus description see: https://youtube.com/watch?v=_4KG0-2ckac ,

Can you provide anything supporting this claim in text version? Crankery existing only in video format tends to be extraordinarily low quality and lame.

Yeah, and Lord Kelvin estimated age of Sun to be about 32 million years (IIRC). Noone claims that scientists are always right.

You're missing the point, perhaps deliberately?

The question centers around why experimentation is important. Anyone can observe something and then make a model up to explain that observation. This does not, and cannot, demonstrate the model is correct. Scientists were wrong about the surface temperature of Venus before it was measured -- yet they made models that perfectly predicted their (incorrect) surface temperature. That their model matched their prediction did not corroborate the model in any way, as is obvious by the fact that it was wrong.

Since then, scientists have measured the temperature of Venus. And now... they have made models to perfectly predict that (correct) surface temperature. Because this time around the temperature is correct, it feels like the model is thus more correct (on this basis) than the previous one. In a sense it is, in that it gives the right temperature. But it is no more (or less) validated by this than the incorrect model! The evidentiary value is exactly the same. You can always make a model fit certain data points, it doesn't mean the model is correct.

Obviously, since we learned about actual Venus temperature any models are expected to predict correct results there.

Doing anything else would be deeply silly.

Specifics how models are build/used/validated are depending on a model. But not rejecting reality and what we learned is hardly indictment of science.

I'll second the request for something not a video.

I've collected these links as well, albeit they are more technical than the video. Will I next be asked to provide a simpler one? :)

  1. "Venus: No Greenhouse Effect" https://theendofthemystery.blogspot.com/2010/11/venus-no-greenhouse-effect.html
  2. "Why does this simple equation predict the Venus surface temperature so accurately?" https://physics.stackexchange.com/questions/508573/why-does-this-simple-equation-predict-the-venus-surface-temperature-so-accuratel

Looking around a little online, I found some people arguing online that of course temperature and pressure make sense together, by the ideal gas law. But they were saying that this doesn't suffice to say that pressure suffices to explain the temperature, as it could be (for example) that temperature affects pressure, rather than the other way around.

What is your evaluation of that argument?

I would say observe that whenever a gas is compressed, the result is both higher pressure and temperature. Gravity compresses a gas as it pulls it to the ground, so this will of course heat it up as well as increasing the pressure.

"Why does this simple equation predict the Venus surface temperature so accurately?" https://physics.stackexchange.com/questions/508573/why-does-this-simple-equation-predict-the-venus-surface-temperature-so-accuratel

See answers posted to that question?

Also, how it even relates to how supposedly "adiabatic lapse rate" can explain Venus' temperature being much higher than would be predicted from blackbody equilibrium?

adiabatic lapse rate here is effect of GHE on Venus. If GHE does not exist, why Venus is much hotter than blackbody equilibrium would predict?

The adiabatic lapse rate falls naturally out of the force of gravity, and non-radiative properties of gases. For the dry adiabatic lapse rate it's actually just the strength of gravity and the heat capacity of the air. You can find a derivation here: http://www.atmo.arizona.edu/students/courselinks/fall10/atmo551a/AdiabaticLapseRate.pdf .

For the moist rate you have to factor in phase-change considerations of the water. This decreases the rate, i.e. the air cools more slowly when water is involved.

Any GHE would have to be on top of/in addition to this. But if the adiabatic lapse rate alone nearly perfectly explains Venus's temperature distribution...

I found those readable enough. Thanks!

After admonishing me for comparing model predicted temperatures of Venus to observation, you link me to a video where the adiabatic lapse model is compared to observations, asserts without independent evidence that this fully explains venus' surface temperatures, and what's more tries to generalize this to earth. This is unsound logic by your own argument.

In any case, I would appreciate it if you would explain your position to me in text, here, rather than sending me links. Or at least provide additional commentary along with the link. After watching that video I am no closer to understanding how adiabadic lapse rate results in surface temperatures in excess of blackbody nor why I should favor this over the greenhouse effect.

Sure. The grand canyon is a good starting point. The temperature at the bottom of the canyon is hotter than at the top. Why is that? It's not due to the greenhouse effect. It's due to earth's adiabatic lapse rate.

Essentially, gravity pulls air in the atmosphere downwards, doing work to compress it, which increases its pressure and temperature. The hotter air then starts expanding and rising (being displaced by the cooler air being brought down), which causes it to cool and decrease in pressure. This is an ongoing process. Notably, it has nothing to do with any radiative properties of the atmosphere (i.e. the greenhouse effect). It can be calculated from basic values of the mass of air and gravity: https://phys.libretexts.org/Bookshelves/Thermodynamics_and_Statistical_Mechanics/Heat_and_Thermodynamics_(Tatum)/08%3A_Heat_Capacity_and_the_Expansion_of_Gases/8.08%3A_Adiabatic_Lapse_Rate .

The lapse rate is essentially the same on Venus as on Earth, and as Venus's atmosphere is thicker than Earth, the lapse rate has a longer way to go, resulting in a higher temperature increase. It must be noted the pressure on Venus's surface is 90x that of Earth's.

The blackbody calculation presumes no atmosphere and thus no adiabatic lapse rate. The presence of an atmosphere and gravity introduces this mechanism by which work is done, heating the air as it compresses and gets close to the surface. It explains the tropospheric temperature gradient and, it must be re-iterated, has nothing to do with any radiative properties of the air. Any atmosphere, even one without any greenhouse gases whatsoever, would have this feature.

The question then is: as the adiabatic lapse rate explains the grand canyon temperature difference, why would it not also explain the temperature difference between the surface and the effective blackbody temperature? It must be noted the effective temperature of Earth (255K, -18C) is indicative of the average amount radiated by an entire column of surface plus atmosphere above. As we've established there must be a gradient due to the lapse rate, the average of this column must necessarily be somewhere in the middle. Below is hotter, above is cooler.

Sure. The grand canyon is a good starting point. The temperature at the bottom of the canyon is hotter than at the top. Why is that? It's not due to the greenhouse effect. It's due to earth's adiabatic lapse rate.

Essentially, gravity pulls air in the atmosphere downwards, doing work to compress it, which increases its pressure and temperature. The hotter air then starts expanding and rising (being displaced by the cooler air being brought down), which causes it to cool and decrease in pressure. This is an ongoing process. Notably, it has nothing to do with any radiative properties of the atmosphere (i.e. the greenhouse effect). It can be calculated from basic values of the mass of air and gravity:

This doesn't make any sense to me. The adiabatic lapse rate describes how the temperature would change if you took a parcel of air, did not allow it to exchange heat (that's the adiabatic part, right?) and moved it up or down so it expands or contracts. As pressure increases or decreases, so does temperature.

But in the Grand Canyon, if gravity is pulling cooler, denser air down, and letting warmer, less dense air rise (as must happen), that's going to result in a cooling effect, not a warming effect. Yes, the cooler air may get a bit more compressed as it falls, and thus rise a little in temperature, but you're also losing warm air that was even warmer when it was at the same altitude, so air circulation would result in a net loss of heat. If you have two regions of air at the same altitude and one is warmer, it will have a rising force compared to the other. Gravity can't make it fall relative to the other one. (To be precise, they could both be rising or falling, just that the cooler one will always fall relative to the warmer one, unless there's momentum of air coming in from outside the system and interacting with the geometry of the landscape, like winds blowing across the canyon).

Gravity is not pulling air downward in a thermodynamics-violating way. If we started out with an atmosphere that was not in steady state, where it was a lot more diffuse and bigger than it should be, then yes, as gravity pulled it down and compressed it, it would get warmer. But that would only happen once (or rather it would oscillate like a spring for a while but eventually settle down).

So yeah, I don't get this at all. I don't know if the temperature gradient at the Grand Canyon is completely due to the greenhouse effect, but I'm pretty sure it's not anything to do with what you're saying, unless I'm misunderstanding you.

Consider the air at the elevation level at the top of the grand canyon. The air that is at ground level at this elevation (eg past the top rim of the canyon) will have a certain temperature. If the grand canyon didn't exist but were equally flat with this ground level, the air there would be the same temperature, right? This is the equilibrium at that height.

Now bring the grand canyon back into existence and allow that air to fall. What happens? As it falls, gravity compresses it, and thus heats it up. By the time it reaches the ground it will be hotter. On its way down, this falling air will displace the air further below it, causing that air below to rise and, due to the lower pressure, expand and cool on its way back up. Thus you have a circulating effect, with the equilibrium temperature increasing with depth.

It doesn't violate thermodynamics as gravity is doing work on the gas, converting potential energy to kinetic energy and increasing its temperature on the way down, while via buoyancy pushing the lower, warmer air up. With no further energy inputs the whole column of air would gradually cool (and eventually freeze and fall out of the sky), but the sun provides the "seed" energy by warming the surface which then warms the air via conduction & convection.

Without the lapse rate basically all ground-level air at any elevation would be the same temperature, the temperature achieved by the sun's warming -- with perhaps mountains slightly warmer as they are closer to the Sun. But the lapse rate additionally causes this effect of warmer air below and cooler air above.

You don't have to take my word for it! Some links:

  • "You can thank a weather phenomenon called adiabatic heating. As air sinks down into a lower elevation, it gets compressed, compressed air releases heat as energy. This caused the air mass to become even warmer.".

    https://edition.cnn.com/2020/06/24/weather/arizona-california-heat-forecast-grand-canyon-shoes-trnd/index.html

  • "In adiabatic cooling, when a mass of air rises—as it does when it moves upslope against a mountain range—it encounters decreasing atmospheric pressure with increasing elevation. The air mass expands until it reaches pressure equilibrium with the external environment. The expansion results in a cooling of the air mass.

    With adiabatic heating, as a mass of air descends in the atmosphere—as it does when it moves downslope from a mountain range—the air encounters increasing atmospheric pressure. Compression of the air mass is accompanied by an increase in temperature."

    https://www.encyclopedia.com/science/encyclopedias-almanacs-transcripts-and-maps/adiabatic-heating

  • "Air molecules play a pivotal role in temperature variation with elevation. When at a low elevation, there are more air molecules compressed together due to the weight of the atmosphere pressing down. As these air molecules are compressed, they generate heat, leading to a temperature increase. Conversely, as elevation rises, air molecules spread apart due to decreased atmospheric pressure, leading to a temperature decrease."

    https://science.howstuffworks.com/nature/climate-weather/atmospheric/question186.htm

The question then is: as the adiabatic lapse rate explains the grand canyon temperature difference, why would it not also explain the temperature difference between the surface and the effective blackbody temperature?

No, sorry, a rhetorical question is not an argument. For the second time, you are still doing the thing you accuse your opponents of: positing that some effect is explained fully by your own pet model without providing any independent evidence that it does so.

It must be noted the effective temperature of Earth (255K, -18C) is indicative of the average amount radiated by an entire column of surface plus atmosphere above.

Are you or are you not trying to rule out that radiative heat transport is a significant factor in atmospheric temperature?

If you neglect radiative heat transport then atmosphere temperature can only ever be less than surface temperature, which is blackbody.

On the other hand, if you include radiative heat transport, then you must acknowledge that different gasses have different absorption/emission spectra and so their behavior cannot necessarily be compared on 1-1 ( or equal density) basis.

No, sorry, a rhetorical question is not an argument. For the second time, you are still doing the thing you accuse your opponents of: positing that some effect is explained fully by your own pet model without providing any independent evidence that it does so.

By saying so you are endorsing the point of the article, which is that this isn't sufficient evidence. I agree. I would certainly never advocate spending trillions of dollars on global projects on this basis without doing further research. Yet that's precisely what the climate alarmists want us to do with their "pet model"s.

And trillions is not an exaggeration! "Without creating the conditions for the massive engagement of the private sector, it will be impossible to move from the billions to trillions that are needed to achieve the SDGs.", said by the Secretary General of the UN in 2023: https://www.un.org/sg/en/content/sg/statement/2023-01-18/secretary-generals-remarks-the-world-economic-forum .

Until then, I can just point out that we have two mutually exclusive explanations, that can't both be right, and insufficient experimental evidence to say which is the correct one. Further on the GHE side of it we have a supposedly powerful physical effect with no experimental (and thus causal) proof that it exists (despite all other physical effects being able to be demonstrated experimentally, even gravity with the Cavendish experiment). And on the adiabatic lapse rate side of it we have rock-solid proof that this is how Earth's atmosphere actually does operate (it does have a lapse rate, the dry rate and moist rates essentially perfectly line up with the rates computed from first principles, etc) and thus it must necessarily also operate in Venus's atmosphere (as physics is universal and works the same everywhere).

It must be noted the effective temperature of Earth (255K, -18C) is indicative of the average amount radiated by an entire column of surface plus atmosphere above.

Are you or are you not trying to rule out that radiative heat transport is a significant factor in atmospheric temperature?

Hmm... so obviously the only way the Earth as a system loses energy is to space via radiation. The "effective temperature" isn't actually a physically real temperature but rather the temperature corresponding to a hypothetical blackbody that would have the same emission as the average radiative emission of Earth to space. And obviously an entire column of surface plus air above it, is what will as a whole be radiating to space.

The question of whether the radiative heat transport warms the surface past the blackbody temperature is separate from the above considerations.

If you neglect radiative heat transport then atmosphere temperature can only ever be less than surface temperature, which is blackbody.

You're leaving out the entire rest of the atmosphere: conduction, convection, water, moisture, latent heat, phase changes, winds, adiabatic lapse rate, etc. etc.

The moon's effective blackbody temperature is the same as Earth's, -18C. Yet it gets to +120C during the day and -120C at night. It's both much hotter and much colder than Earth and than the effective blackbody temperature.

The entire atmosphere participates in the redistribution of this heat, to be cooler during the day and warmer at night. Not just the tiny percent that absorbs and emits infrared radiation.

By neglecting all that and leaving only one option, radiation, of course your thoughts will naturally be directed towards assuming and thus believing that it must account for everything. But you leave out all the rest.

Not to mention that by considering the effective blackbody temperature, you're considering an average and also neglecting the fact that there's day and night, that the Sun warms the planet more on its day side than night side, etc.

On the other hand, if you include radiative heat transport, then you must acknowledge that different gasses have different absorption/emission spectra and so their behavior cannot necessarily be compared on 1-1 ( or equal density) basis.

Their non-radiative effects can be. All that is needed for adiabatic lapse rate is to have mass, heat capacity, and gravity. CO2 accomplishes this as well as any other gas.

As to what effect the differing radiative properties have (which differing properties they do have), that is indeed what's under discussion here.

As far as I can see, you still have not given any explanation for how the lapse rate effect can result in temperatures far in excess of blackbody (day/night temps being irrelevant since we are interested in average temperatures)

Take a packet of gas that starts at the surface, rises to its maximum height, and then falls back to the surface. Initially it will be in equilibrium with the surface temperature. If the gas does not absorb or emit significant radiation, then it will have the same temperature at the end of the round trip as the start. There is still no mechanism by which the gas packet temperature would exceed the surface temperature nor by which surface temperature would exceed blackbody.

And obviously an entire column of surface plus air above it, is what will as a whole be radiating to space.

If a packet of gas does not exchange (absorb or emit) significant energy via radiation then the "whole column of air" will not transfer energy to space.

More comments

I think a more compelling argument might be "back in the 1970s the Science said that we were heading for an Ice Age due to industrial pollution and emissions - plus they've been predicting catastrophic climate change (melting of the ice caps, massive temperature changes and disruption) by 2000, 2010 and 2020 so they clearly don't know what they're doing'. We don't need to get bogged down in whether the greenhouse effect works as stated.

https://www.aei.org/carpe-diem/50-years-of-failed-doomsday-eco-pocalyptic-predictions-the-so-called-experts-are-0-50/

I'm not sure why that should be compelling at all. "Science" isn't just one coherent entity where one scientist being wrong makes all of the rest in the vaguely same sector fundamentally wrong. One scientist, who might as well not even be alive any more, making a prediction in 1967 has no bearing on scientists making predictions right now. Much of the list isn't even concerned with scientists - neither Al Gore or Prince Charles are such - or is related to issues other than climate change, such as peak oil, which has plenty of advocates as a theory who don't consider climate change to be all that dangerous (Greer, say).

"Wow! Look at all these failed AI predictions!" is a lackluster argument in debates about when the AGI is coming, if it is at all, and this is similarly a lackluster argument in climate debates.

With AI narratives they can point to trends and those trends are undeniably real! Nobody said 'oh AGI is impossible because the number of transistors on a chip is decreasing, we're actually heading for artificial stupidity', that would be silly. Furthermore, AI development is fundamentally unpredictable, people talk about probabilities of developing certain technologies by certain times.

Climate change is supposed to be a clear physical-material trend, yet there's been confusion about which direction it's moving and what consequences there might be and when they arrive. If they could get it wrong in the 1960s, 1970s, 1980s, 1990s and 2000s they can get it wrong today. The Pentagon report for instance:

https://www.theguardian.com/environment/2004/feb/22/usnews.theobserver

A secret report, suppressed by US defence chiefs and obtained by The Observer, warns that major European cities will be sunk beneath rising seas as Britain is plunged into a ‘Siberian’ climate by 2020. Nuclear conflict, mega-droughts, famine and widespread rioting will erupt across the world.

The document predicts that abrupt climate change could bring the planet to the edge of anarchy as countries develop a nuclear threat to defend and secure dwindling food, water and energy supplies. The threat to global stability vastly eclipses that of terrorism, say the few experts privy to its contents.

Among those scientists present at the White House talks were Professor John Schellnhuber, former chief environmental adviser to the German government and head of the UK’s leading group of climate scientists at the Tyndall Centre for Climate Change Research. He said that the Pentagon’s internal fears should prove the ‘tipping point’ in persuading Bush to accept climatic change.

Bob Watson, chief scientist for the World Bank and former chair of the Intergovernmental Panel on Climate Change, added that the Pentagon’s dire warnings could no longer be ignored.

Randall added that it was already possibly too late to prevent a disaster happening. ‘We don’t know exactly where we are in the process. It could start tomorrow and we would not know for another five years,’ he said. ‘The consequences for some nations of the climate change are unbelievable. It seems obvious that cutting the use of fossil fuels would be worthwhile.’

These weren't no-names or non-scientists but they were seriously and embarrassingly wrong. Imagine if we actually listened to these people, speedily cut fossil fuels out of the world economy accepting the energy rationing, economic mobilization and famines that would likely happen... only for it to be a nothingburger.

In AI there are clear achievements and errors on both sides - some people said we'd get self-driving vehicles by 2000, others by 2030, others never. Well, we have self-driving cars today. We have GPT-4. Things the AI alarmists foresaw actually showed up, perhaps not on time but they did arrive!

These weren't no-names or non-scientists but they were seriously and embarrassingly wrong. Imagine if we actually listened to these people, speedily cut fossil fuels out of the world economy accepting the energy rationing, economic mobilization and famines that would likely happen... only for it to be a nothingburger.

No they weren't, The Guardian just made that up. It's not a prediction, it's a brief outline of a hypothetical written by two non-scientists (both self-professed futurists working for the consulting firm Global Business Network) who specifically state that it is extreme and unlikely. The point is not that they think it is likely to happen, but that they think such unlikely but extreme scenarios should be considered and prepared for by the Pentagon.

An Abrupt Climate Change Scenario and Its Implications for United States National Security

We have created a climate change scenario that although not the most likely, is plausible, and would challenge United States national security in ways that should be considered immediately.

These are the steps they propose be taken:

  • Improve predictive climate models to allow investigation of a wider range of scenarios and to anticipate how and where changes could occur
  • Assemble comprehensive predictive models of the potential impacts of abrupt climate change to improve projections of how climate could influence food, water, and energy
  • Create vulnerability metrics to anticipate which countries are most vulnerable to climate change and therefore, could contribute materially to an increasingly disorderly and potentially violent world.
  • Identify no-regrets strategies such as enhancing capabilities for water management
  • Rehearse adaptive responses
  • Explore local implications
  • Explore geo-engineering options that control the climate.

Notice that reducing CO2 emisssions isn't even mentioned because their scenario is so abrupt that it would be too late, rather they are talking about preparing ways to mitigate the damage and/or do emergency geo-engineering, in case an unlikely scenario like that happens.

This report suggests that, because of the potentially dire consequences, the risk of abrupt climate change, although uncertain and quite possibly small, should be elevated beyond a scientific debate to a U.S. national security concern.

Among those scientists present at the White House talks were Professor John Schellnhuber, former chief environmental adviser to the German government and head of the UK’s leading group of climate scientists at the Tyndall Centre for Climate Change Research. He said that the Pentagon’s internal fears should prove the ‘tipping point’ in persuading Bush to accept climatic change.

Bob Watson, chief scientist for the World Bank and former chair of the Intergovernmental Panel on Climate Change, added that the Pentagon’s dire warnings could no longer be ignored.

So scientists find a speculative report by non-scientists and trumpet it as 'this is why you should reduce Co2 emissions'. It's just like the IPCC reports. They're tremendously dull documents that say things like 'high confidence that water stress will moderately increase in the medium term' and don't mention any existential threat except to Pacific islands. Meanwhile Extinction Rebellion is screeching about the apocalypse. Meanwhile governments are signing legal targets that commit them to deindustrialization and high energy costs, shutting down farms and so on.

There's a gap between what actual scientists are saying and what the message flowing through to policymakers, celebrities, media and the public is. Rather like COVID, the real science on mass-scale mask use was mixed and unclear, yet the Science mandated them.

James Hansen is/was screeching about disaster, the Arctic was supposed to melt about 10 years ago: https://news.google.com/newspapers?nid=1988&dat=20080624&id=7mgiAAAAIBAJ&sjid=7qkFAAAAIBAJ&pg=5563,4123490

I can't comment on most of this, except the following:

This makes it all the more peculiar that nobody has been able to experimentally demonstrate and therefore verify the greenhouse effect.

Wouldn't the existence of Venus be pretty definitive proof that such a thing is possible?

Venus has twice the irradiance, 90 times the atmosphere, no significant phase change cycles, no water to absorb energy, slow diurnal rotation. Just do a comprehensive experiment on Earth, it will do us more good.

This makes it all the more peculiar that nobody has been able to experimentally demonstrate and therefore verify the greenhouse effect.

Experimentally proving the greenhouse effect is a common science class task for sixth graders. The author of this could probably do it with materials that exist in his house. This is a rather extreme self-imposed ignorance.

The ignorance is coming from this reply not from the author of the article. It is salient that you didn’t actually provide a link, source, or explanation of how the GHE can thus be verified. It also doesn’t address why the referenced 2021 peer reviewed paper said a lab verification was lacking — because the typical demonstrations (involving gas in glass jars or plastic bags) don’t demonstrate the GHE. Their results are an effect of gas densities affecting convective heat loss, not radiative effects. They work equally well with Argon.

They work equally well with Argon.

Is it based on experiment or a guess?

On experiments:

  1. "Benchtop Global-Warming Demonstrations Do Not Exemplify the Atmospheric Greenhouse Effect, but Alternatives Are Available", https://pubs.acs.org/doi/10.1021/acs.jchemed.8b01057

    _

    (It must be noted the "alternatives" provided are analogues, not demonstrations of the GHE.)

  2. "Climate change in a shoebox: Right result, wrong physics", https://www.researchgate.net/publication/243492513_Climate_change_in_a_shoebox_Right_result_wrong_physics .

Here is a benchtop experiment. Figure 6 apparently shows that variations in the density of air in a balloon do not affect it's cooling rate, whereas it does for a CO2 filled balloon. This would seem to contradict your claim that it's a spurious density effect.

https://royalsocietypublishing.org/doi/10.1098/rsos.192075

It may be worth reviewing the links I just provided as to why Argon is a valuable control for these experiments (https://www.themotte.org/post/960/the-vacuity-of-climate-science/203988?context=8#context).

In short: carbon dioxide is more dense than air. CO2 and Argon are about the same density as each other. A typical experiment involves adding CO2 to a beaker and observing the surface temperature get hotter. But the reason is that the convective loss of the bottom is suppressed because the air is prevented from rising due to the heavier CO2 above it. This works equally well with Argon. In closed containers, the differing gas density still affects the rate of convective loss, as #2 points out.

The experiment you linked must therefore also be performed with 100% Argon (as it was with 100% CO2) as a control.

I dont see how those are relevant to the study I posted. Those were open systems where mass exchange with the environment is possible, but in the one I posted the gasses are sealed in balloons.

This was the relevant part: "In closed containers, the differing gas density still affects the rate of convective loss, as #2 points out."

Quoting from the paper, emphasis added:

Even if the experiments were done in a sealed container, and CO2 were compared with Ar, differences in convective transport could not be ruled out.

[...]

In some of the reported demonstrations, the container is sealed so that interfacial mixing should not be a factor, but a temperature difference between air and CO2 was still reported.[3–5] Sealing the container brings into play additional factors, which are difficult to quantify, especially the infrared characteristics of the “window.” In Ref. 5, for example, T is reported to be 25% greater for CO2 than for air. As we have argued, a difference of this magnitude cannot be accounted for by far-infrared absorption. This experiment probably involves the direct absorption of sunlight rather than of reemitted far-infrared radiation, but that effect should be even smaller. [7] Simple models of natural convection suggest that the difference in the heat flow between the two gases should be less than 10%.[17] The calculations depend on the geometry, however, and the conditions are in a transition regime between laminar and turbulent flow, and thus it is possible that small changes could have disproportionate effects. These uncertainties reinforce the value of including a comparison case that uses a heavy gas without infrared absorption, such as argon.

The key here is “experimentally demonstrate”. Pointing to Venus isn’t an experiment! I thought the following demonstrated why rather effectively:

“The problem this poses is best exemplified by going back to Pekeris 1932. (It must be noted that Hansen et al 1983 cites Wang 1976 which cites Goody 1964 who then cites Pekeris 1932). In Pekeris, the models of the time led the scientist to believe that “it becomes plausible the temperatures on […] Venus, Earth, and Mars are about the same”. As Venus’s temperature is 464ºC while that of Mars is -63ºC, his egregious error reveals the fundamental problem of not having experimental means by which to validate models. This leads to a situation where models are susceptible to overfitting available data, with no ability to check their operations by proving that the actual effect matches the model’s prediction.”

Sure, it’s an observation, rather than a prediction. That’s fine for a control group, but insufficient. At least it lets us rule other theories out.

So…what would constitute experimental demonstration? What could I do to convince you that the greenhouse effect can, in fact, trap some amount of heat?

For that matter, I don’t see you providing any experiments yourself! Why should I privilege your bench-top reasoning over the IPCC’s?

It’s a good question and is already addressed in the article:

“Although the burden of proof is on a theory’s proponents rather than its critics, we can conjecture what one such proof might look like: it would have to consist of an external energy source – such as the sun or a heat lamp – that is set to warm a surface. The energy input should be measured and the surface, in the presence of greenhouse gases, should get much hotter than that input alone can provide, emitting much more energy in response. This would definitively demonstrate the greenhouse effect itself, after which the anthropogenic influence could be gauged by introducing more carbon dioxide into the apparatus and measuring the marginal temperature response.”

From your first Twitter link, the guy gets it wrong right off the bat. "How can the GHGE work after so long a night?" Because of the thermal mass of the ground and atmosphere, and the insulating properties of the atmosphere which is much thicker than Earth's, and because high winds on Venus tend to equalize day/night side temps.

This guy doesn't know what he's talking about.

https://twitter.com/_Escapekey_/status/1748317807006118335 is also cute, as growth is also visible before their claimed start of conspiracy (even on their own graph, no idea is any claim there even accurate)

Thanks!

From your second link:

A typical validation of the Stefan-Boltzmann’s law of radiation : is done by measuring the radiation from a filament of the incandescent lamp. The filament is enclosed in a vacuum.

A typical demonstration of radiative heat transfer is done in a vacuum, but that isn't a real requirement. You can instead compare the total heat transfer from a hot black plate (emissivity near 1) to the heat transfer from a hot silver plate (emissivity near 0). The black plate will have faster heat transfer despite being surrounded by the same air because it radiates more.

I'd like to see his calculations for "...the conductive and convective effects at the surface are vastly greater than the radiation; by about 240 times."

Venus has a very high albedo. Most sunlight is actually reflected away from the planet. Venus actually gets less net sunlight than earth! GHGE says Venus, without a greenhouse gas effect, should be cooler than earth!

No!!

Even if you assume the emissivity of the object (such as Venus) can be fully described in a single number (i.e. it is an ideal gray body), you're only describing the gross rate of radiative heat transfer. Any ideal gray body that was protected from conduction/convection would reach the same equilibrium temperature given the same surroundings; a high-emissivity one would absorb a lot of energy which is coming in and emit just as much, while a low-emissivity one would absorb a tiny bit of energy and emit just as little.

It's a little weird that the author repeatedly denies any experimental verification of the greenhouse effect but doesn't address the 19th century experiments that first identified it. Maybe there's something wrong with his results, but you wouldn't know it from reading this post.

I had a very negative reaction to this article. I think it reads much better with the following set up to contextualize it.

Start by contemplating the power of bench top science and theory. The partnership of bench top science and theory has some spectacular successes to its credit. You can experiment with Newtonian mechanics in your laboratory, verifying the basic laws. Then you get a top mathematician (Euler) to work out what those laws imply for gyroscopes. Later engineers build a gyrocompass for a submarine. Will this really work? Underwater? A thousand miles from the laboratory? Yes!

Or think of James Clerk-Maxwell, taking the bench top science of Ampere and Faraday and coming up with Maxwell's equations. The equations predict electromagnetic radiation. Hertz does the experiments in his laboratory and finds them; a great triumph for theory. Later Marconi takes this out into the real world. Theory shows that electromagnetic radiation goes in straight lines; Marconi's attempts at radio communication beyond the curve of the earth are not going to work. And sure enough they fail. Wut! Marconi actually succeeds! But rather than concede that there are problems getting out of the laboratory and into the real world, we discover the ionosphere and chalk it up as another spectacular success for the partnership of bench top science and theory.

Move on to contemplating the contrasting situation in medicine. The human body is too complicated for the human mind to comprehend. Basing medicine on theory works badly for the patient, no matter how much money it brings in for the doctor. This has lead to evidence based medicine. Never trust the combination of lab bench chemistry and theory. Always do a randomised controlled trial to check that medicines really work. Theory said that vitamin E was an anti-oxidant and would reduce oxidative stress and prevent cancer. Epidemiology confirmed this. Randomised controlled trials refuted it. Examples are so numerous that you can fill a book.

Returning to climate science, we must ask whether it is like gyroscopes or like Vitamin-E/Vioxx/vertebroplasty. Doing bench top science with an infrared spectroscope and a sample of Carbon Dioxide yields uncontroversial results. But does it have implications for the weather?

I've picked up the impression that every-one agrees on the importance of feedback loops. If you believed that climate science was a like a gyroscope, you would compute warming on the basis of the infrared characteristics of Carbon Dioxide and conclude that we are in for some warming, but not enough to constitute a crisis. No-one believes that climate science is like a gyroscope. Some think that warmer air means more water vapor which means more warming and more clouds and more clouds mean mumble. Subtle feedback in the atmosphere is putting us on a course for disaster. Others disagree.

The article emphasizes one particular disagreement. Scientists attempt empirical confirmation of the theory, but they mess it up. All the empirical work is heavily contaminated by theory. It cannot refute the theory because it assumes the theory.

For EM, straight-line propagation was a null hypothesis. Practical experiments rejected it, so the theory had to adapt. Science worked as intended.

For medicine, it’s a little trickier, because the category is defined by passing RCTs. The null hypothesis for any given treatment is that it doesn’t work; only those which can reject that are allowed into the category. Then anything which gets counted as effective must have some practical experiments behind it.

Flat Earthers object to the null hypothesis of, uh, Round Earth. In 2018, there was a documentary going around where one of them set up a practical experiment. Predictably for the rest of us, he failed to reject the hypothesis.

What’s the null hypothesis for climate skepticism?

The situation is different from EM or medicine because skeptics are unable to provide practical evidence of their own. After all, they’ve got all the same constraints as the IPCC—preexisting data, lack of a control group—but with less funding and less experience. Until they can move up a level and cite a practical experiment, they’re going to be stuck with the same kind of arguments as their opponents.

The flat earth documentary was fascinating because the guy actually proved the Earth was round in his experiment. Then they had to explain away their results and pretend it didn't happen.

Re: climate science, I think the situation is the same as with medicine. The default assumption on medicine is a new thing doesn't work. So you have to positively prove it. Only then can your intervention be recommended. For climate policy, it should be the same. You want to impose new taxes, de-industrialize (which industrialization has massively benefited humanity), ban synthetic fertilizer (which synthetic fertilizers are responsible for vast amounts of our food production), degrow the economy, etc? Then you have to actually prove, definitively, there is an issue, and that your policy would work.

This is not the state of affairs today. Today it's just presumed that the climate alarmists are right. If you question it they say "the science is settled" and smear you as a "denier" (which grew out of the "Holocaust denier" term). They've flipped the burden of proof in the public and policy sphere. But that's not how science is done. The person proposing the theory is the one that needs to prove it. It's not up to others to falsify it.

That being said I'm working on an experiment that could actually definitively falsify the GHE. Will see how it goes!

Out of curiosity, what's the experiment?

Essentially a repeat of Wood's 1909 experiment (http://www.wmconnolley.org.uk/sci/wood_rw.1909.html), except actually measuring the solar insolation and the backradiation with a pyranometer and pyrgeometer, respectively. Then experimentally measuring heat losses due to conduction and convection. Finally seeing if these add-up as they should on the different setups (IR-opaque glass vs IR-transparent rock salt). So we'll see definitively whether added backradiation causes warming or not.

Hi, the author does address it :)

“Origins of the Theory

If the greenhouse effect has never been demonstrated, then why does anybody believe that CO2 causes warming in the first place? The answer is somewhat embarrassing. The first to propose the effect was Fourier in 1824, who (according to Arrhenius) believed that an actual greenhouse works “because it lets through the light rays of the sun but retains the dark rays [i.e. infrared radiation] from the ground”. Fourier proposed the atmosphere had a similar property. When Tyndall discovered in 1859 that CO2 does indeed absorb infrared radiation, early climate modelers took this fact and ran with it, with Arrhenius doing “the first calculation of the warming of Earth due to CO2 increase” in 1896. But actual greenhouses do not work this way. They work by physically blocking hot air from escaping and mixing with the outside air. This was shown by R W Wood as early as 1909. It is more than strange that despite this falsification, and a lack of any further proof, the theory has persisted to this day.”

The key is that Tyndall showed CO2 absorbs IR, but this in and of itself isn’t the greenhouse effect. The effect is the (extra) warming a surface undergoes as a result of said absorption.

I don't get it.

The key is that Tyndall showed CO2 absorbs IR, but this in and of itself isn’t the greenhouse effect. The effect is the (extra) warming a surface undergoes as a result of said absorption.

If you agree that CO2 absorbs IR, and therefore an atmosphere with more CO2 gets warmer than an atmosphere with less CO2 given a constant heat input (as far as I can tell this is experimentally verified), I don't see what there is to discuss.

The digression about how greenhouses don't work this way appears to be totally irrelevant to the question of whether the greenhouse effect exists. A rose by any other name etc etc.

The greenhouse diversion is unfortunate due to the naming but also interesting. We can call it a hothouse instead and call the way it really works the “hothouse effect”, which is suppression of convection. HHE != GHE , of course, so it’s not relevant on that level.

What’s interesting to consider though is that a hothouse made of glass vs a hothouse made of a thin plastic, works essentially as effectively. But the glass absorbs and thus emits more IR, both into and out of the hothouse. This downward IR should, by the GHE, warm the interior much more than the plastic walls which don’t emit nearly as much IR. But, it doesn’t. This is peculiar and points to the GHE being weak rather than powerful as it’s said to be (causing the Earth to be +33°C warmer than it otherwise would be).

This is peculiar and points to the GHE being weak

So you are admitting that the GHE exists and we are merely quibbling over effect sizes? Otherwise, I still don't know why we are talking about greenhouses.

Greenhouses are relevant because the GHE should be the mechanism by which they work. But, they don't. That they work due to the HHE and not the GHE is evidence against the GHE.

Not really. You can search and replace "greenhouse effect" with "florb effect" and it won't change climate science one iota.

It also won't change the point here. I'll try it again with newly coined terms in bold.

The florb effect is a radiative phenomenon. Any object above 0K absorbs & emits infrared radiation. Certain gases, like CO2, also absorb & emit infrared radiation, while others don't. If you increase the concentration of CO2 in the air, the air now will be emitting more infrared radiation, both upwards and downwards back toward the surface. This downward emission of infrared causes the surface to become warmer. This warming is called the florb effect.

A hothouse, by contrast, works by the hothouse effect, a totally different phenomenon. The sun heats the inside of the hothouse, which causes the air inside to become warmer. This air rises, but is physically prevented from escaping by the hothouse walls. This allows the air inside the hothouse to get warmer than the outside air. It is an effect of suppressing convective loss rather than a radiative phenomenon.

The peculiar thing: A hothouse remains equally hot whether its walls are transparent to infrared, or absorbing of infrared! This is despite the IR-absorbing walls emitting infrared into the hothouse, while the IR-transparent walls don't. According to the depicted mechanism of the florb effect, the IR-emitting walls should result in a much hotter hothouse interior. Yet, it doesn't.

Thus, the fact that a hothouse works due to the hothouse effect, and not due to the florb effect, is evidence against the florb effect.


You see now?

I think that's a great edit because it makes the non-sequitur in the final paragraph very clear.

More comments

HHE

What is that? Probably you are not meaning Health Hazard Evaluation.

"The greenhouse diversion is unfortunate due to the naming but also interesting. We can call it a hothouse instead and call the way it really works the “hothouse effect”, which is suppression of convection. HHE != GHE , of course, so it’s not relevant on that level."

HHE = the "hothouse effect" I just coined here for clarity in communication.

It’s a bit subtle although not really.

The GHE is actually that the surface gets warmer, not the atmosphere. Actually the lower atmosphere (the troposphere) is warmed far more by conduction and convection than radiative effects. The 0.04% quantity of CO2 actually is very relevant here, even if that amount it absorbed doubled, it’s still far outclassed by convection which 100% of the air participated in.

But that’s a furphy as far as GHE is considered, because GHE is really about the extra downward emission of IR, not the atmosphere itself warming due to absorption. This extra downward IR is said to warm the surface more, given same energy input.

This latter piece is what hasn’t been demonstrated and what some physicists argue is thermodynamically impossible. I don’t want to debate the theory of that (most say it’s possible, a vocal few say impossible; not much we can add to it ourselves here), I’d rather defer to experimental evidence, and it being lacking doesn’t look good for the “it’s possible” side.

If the atmosphere is warmer, it seems straightforward that the ground becomes warmer than it would be if the atmosphere was colder.

That would be not due to the GHE (which is radiative) but something else, like say convection loss being less because of warmer atmosphere. If this were the case it would mean all the climate models are wrong, because they all rely on the GHE (downwards IR causing higher surface temps). I'm happy to accept this and then we could talk about what that might look like.

Two points though:

  1. In the troposphere the CO2 absorption is already saturated, as Knut Angstrom's assistant found in 1900. So the effect on the lower atmosphere would essentially be nil

  2. The alarmists' rebuttal to #1 is that higher up in the atmosphere, at colder and lower-pressure layers, the effect is not yet saturated. Perhaps those, then would become warmer? Yet consider that if the air absorbs more IR, so, too, does it emit more IR. Added CO2 provides the air a way to cool that it wouldn't otherwise have. In fact, the standard understanding of the climate consensus is the stratosphere ought to cool with more CO2 rather than warm up.

    So if the troposphere doesn't warm due to more CO2 as its absorption is already saturated, and the stratosphere cools, and the downwards IR doesn't have a warming effect... where does that leave the warming theory then?

I don't know a thing about climate models so I can't answer your questions.

My only point is that you don't actually seem to deny that the greenhouse effect exists and is empirically verified. You seem to be quibbling about how exactly it works and how strong it is.

However, your article was very confident about the much stronger claims which you now, in my view, have walked back. In addition, you seem to evade a lot of questions in this thread and instead link various YouTube videos or otherwise unrelated content.

From this, it appears that you are in some way trying to hide the ball in this debate and therefore I have no choice but to spend many hours researching what is actually going on here or simply discount your claims as untrustworthy and going about my business. I'll go with the second option.

Ehm... at the time you wrote this, I provided two links. One was to another post I made in this thread, and one was to a YouTube video. There are no "various YouTube videos" linked, just one (which was related to the discussion), and no "otherwise unrelated content" at all (in fact, no links to other content at all besides the one video). So it seems you're operating on a mind-already-made-up, prejudiced basis and are interpreting what I'm writing via feelings, emotions, and pre-existing beliefs rather than sensible facts and clear-headed evaluation of what I'm writing here.

Your other points are equally invalid. I haven't evaded any questions -- you'll have to point out where I have if you want to make that claim, but you won't be able to, as I haven't.

I haven't written anywhere that I think the GHE is empirically verified. Rather to the contrary, I've written how it hasn't been experimentally verified (https://www.themotte.org/post/960/the-vacuity-of-climate-science/203568?context=8#context). And no, this is not a link to "otherwise unrelated content", it's a link to something else I wrote in this thread.

You are misinterpreting what I wrote about greenhouses. The point is that, the fact that greenhouses don't work according to the GHE (see https://www.themotte.org/post/960/the-vacuity-of-climate-science/203995?context=8#context if you are getting tripped up on the terminology here) does not prove, in and of itself, that the GHE doesn't exist, but it does speak very much against it. This doesn't mean the GHE exists, it just means the fact of how greenhouses work is not sufficient evidence to disprove the GHE.

Perhaps the only valid thing you did write here is that you "don't know a thing about climate models" (your words). I can accept that. But the salient question then is, whence do you draw the confidence with which you've formed your opinion about what I wrote?

To help you answer that question I will re-iterate something I wrote at the start of the article that is very relevant here, with emphasis added: "This is highly relevant because it means our current climate scare is based not on irrefutable scientific evidence but rather on hysteria and alarmist fear-mongering that fifty years of “failed apocalyptic predictions” have failed to abate. This is crucial to understand as it makes it clear that rather than debating how humans should mitigate this alleged impending disaster, the proper focus should be to question why those in power are employing psychological fear tactics to promote taxation, restriction, and degrowth, and why so many intelligent people have uncritically bought into the hysteria when these proposed policies are clearly to their own detriment."